Closure of the union = Union of closures












3














I have seen that $text{cl}(Acup B)=text{cl}(A)cup text{cl}(B)$. However I don't see why this is true. I can see the the right to left inclusion, but I can't see the inclusion from left to right. Say that I have an element $x$ contained only in two open sets one that intersects $Acup B$ only in $Asetminus B$ and and another that intersects only in $Bsetminus A$ isn't this a contradiction?



Edit: I have seen the proof but I still can't understand what is wrong with the counterexaple above



Thanks in advance










share|cite|improve this question
























  • Choose some limit point of LHS and observe that it belong to the RHS.
    – Masacroso
    Oct 26 '16 at 16:06
















3














I have seen that $text{cl}(Acup B)=text{cl}(A)cup text{cl}(B)$. However I don't see why this is true. I can see the the right to left inclusion, but I can't see the inclusion from left to right. Say that I have an element $x$ contained only in two open sets one that intersects $Acup B$ only in $Asetminus B$ and and another that intersects only in $Bsetminus A$ isn't this a contradiction?



Edit: I have seen the proof but I still can't understand what is wrong with the counterexaple above



Thanks in advance










share|cite|improve this question
























  • Choose some limit point of LHS and observe that it belong to the RHS.
    – Masacroso
    Oct 26 '16 at 16:06














3












3








3


0





I have seen that $text{cl}(Acup B)=text{cl}(A)cup text{cl}(B)$. However I don't see why this is true. I can see the the right to left inclusion, but I can't see the inclusion from left to right. Say that I have an element $x$ contained only in two open sets one that intersects $Acup B$ only in $Asetminus B$ and and another that intersects only in $Bsetminus A$ isn't this a contradiction?



Edit: I have seen the proof but I still can't understand what is wrong with the counterexaple above



Thanks in advance










share|cite|improve this question















I have seen that $text{cl}(Acup B)=text{cl}(A)cup text{cl}(B)$. However I don't see why this is true. I can see the the right to left inclusion, but I can't see the inclusion from left to right. Say that I have an element $x$ contained only in two open sets one that intersects $Acup B$ only in $Asetminus B$ and and another that intersects only in $Bsetminus A$ isn't this a contradiction?



Edit: I have seen the proof but I still can't understand what is wrong with the counterexaple above



Thanks in advance







general-topology






share|cite|improve this question















share|cite|improve this question













share|cite|improve this question




share|cite|improve this question








edited Oct 27 '16 at 15:49









Hermès

1,809612




1,809612










asked Oct 26 '16 at 15:55









TheGeometer

920519




920519












  • Choose some limit point of LHS and observe that it belong to the RHS.
    – Masacroso
    Oct 26 '16 at 16:06


















  • Choose some limit point of LHS and observe that it belong to the RHS.
    – Masacroso
    Oct 26 '16 at 16:06
















Choose some limit point of LHS and observe that it belong to the RHS.
– Masacroso
Oct 26 '16 at 16:06




Choose some limit point of LHS and observe that it belong to the RHS.
– Masacroso
Oct 26 '16 at 16:06










4 Answers
4






active

oldest

votes


















2














(1) ($supset$) ::
begin{align*}
A subset A cup B implies text{cl}(A) subset text{cl}(A cup B)
\B subset A cup B implies text{cl}(B) subset text{cl}(A cup B)
end{align*}
therefore yielding that $text{cl}(A) cup text{cl}(B) subsettext{cl}(A cup B)$



(2) ($subset$) ::



The subset $text{cl}(A) cup text{cl}(B)$ is closed and both contains $A$ and $B$, therefore $A cup B subset text{cl}(A) cup text{cl}(B)$. $text{cl}(A cup B)$ is defined to be smallest closed set which contained $A cup B$, so that any closed set which contained $Acup B$ also contains $text{cl}(A cup B)$. Therefore $text{cl}(A cup B) subset text{cl}(A) cup text{cl}(B)$.






share|cite|improve this answer





















  • Thanks for answer the proof makes sense but I still can't see what is wrong with my counterexample
    – TheGeometer
    Oct 26 '16 at 16:26










  • But how come $x$ in cl(A)? I have a neigbourhood of $x$ only intersecting at BA so this neigbourhood does not intersect A so it shouldn't be a limitpoint of A right?
    – TheGeometer
    Oct 26 '16 at 16:36










  • @TheGeometer Indeed, I've got confused! Define your two open sets to be $U_1$ and $U_2$. Then $U_1 cap U_2$ is a third open set containing $x$, that neither intersect $A$ or $B$. So $x notin text{cl}(A cup B)$ ;). So it's not a counterexample.
    – Hermès
    Oct 26 '16 at 16:44












  • Exactly! Thanks man I got so paranoid with that problem:P
    – TheGeometer
    Oct 26 '16 at 16:45



















2














In your proposed counterexample, you've forgotten that open sets are closed under finite intersection.



So if $x$ has a neighborhood that only meets $Acup B$ in $Bsetminus A$ and a neighborhood that only meets $Acup B$ in $Asetminus B$, then the intersection of these neighborhoods doesn't meet $Acup B$ at all.






share|cite|improve this answer





























    1














    Let $xin Cl(Acup B)$ then every open set containing $x$ intersects $Acup B$. Thus for $x in U_alpha$ where $U_alpha$ is open in $X$. If $U_alpha$ intersects $A$, then $x in Cl(A)$ else $x in Cl(B)$ either way $x in Cl(A)cup Cl(B)$.






    share|cite|improve this answer





















    • This is because if $yin Cl(A)$ iff every open set containing $y$ intersects $A$.
      – Riemann-bitcoin.
      Oct 26 '16 at 16:10










    • I think I have a confusion with the definition. How I know it is that $x$ is a limit point of a subset $S$ of a topological space if every neigbourhood of $x$ intersects with $S$. So in our case if $x$ has a neigbourhood only intersecting $AB$ and another only intersecting $BA$ it should be a limit point only of $AcupB$ and not of A or B.Am I using wrond definition or saying something wrong?
      – TheGeometer
      Oct 26 '16 at 16:19










    • The $Cl(A)$ is the limit points unioned with the set A. So $Cl(A)=A cup A'$. I believe this is the confusion, though I am slightly confused on what your counterexample is saying.
      – Riemann-bitcoin.
      Oct 26 '16 at 16:57





















    0














    Infinite point sequence from (A ∪ B) contains an infinite subsequence from A or contains an infinite subsequence from B






    share|cite|improve this answer





















    • ok but what is wrong wit my counterexample?
      – TheGeometer
      Oct 26 '16 at 16:04











    Your Answer





    StackExchange.ifUsing("editor", function () {
    return StackExchange.using("mathjaxEditing", function () {
    StackExchange.MarkdownEditor.creationCallbacks.add(function (editor, postfix) {
    StackExchange.mathjaxEditing.prepareWmdForMathJax(editor, postfix, [["$", "$"], ["\\(","\\)"]]);
    });
    });
    }, "mathjax-editing");

    StackExchange.ready(function() {
    var channelOptions = {
    tags: "".split(" "),
    id: "69"
    };
    initTagRenderer("".split(" "), "".split(" "), channelOptions);

    StackExchange.using("externalEditor", function() {
    // Have to fire editor after snippets, if snippets enabled
    if (StackExchange.settings.snippets.snippetsEnabled) {
    StackExchange.using("snippets", function() {
    createEditor();
    });
    }
    else {
    createEditor();
    }
    });

    function createEditor() {
    StackExchange.prepareEditor({
    heartbeatType: 'answer',
    autoActivateHeartbeat: false,
    convertImagesToLinks: true,
    noModals: true,
    showLowRepImageUploadWarning: true,
    reputationToPostImages: 10,
    bindNavPrevention: true,
    postfix: "",
    imageUploader: {
    brandingHtml: "Powered by u003ca class="icon-imgur-white" href="https://imgur.com/"u003eu003c/au003e",
    contentPolicyHtml: "User contributions licensed under u003ca href="https://creativecommons.org/licenses/by-sa/3.0/"u003ecc by-sa 3.0 with attribution requiredu003c/au003e u003ca href="https://stackoverflow.com/legal/content-policy"u003e(content policy)u003c/au003e",
    allowUrls: true
    },
    noCode: true, onDemand: true,
    discardSelector: ".discard-answer"
    ,immediatelyShowMarkdownHelp:true
    });


    }
    });














    draft saved

    draft discarded


















    StackExchange.ready(
    function () {
    StackExchange.openid.initPostLogin('.new-post-login', 'https%3a%2f%2fmath.stackexchange.com%2fquestions%2f1986224%2fclosure-of-the-union-union-of-closures%23new-answer', 'question_page');
    }
    );

    Post as a guest















    Required, but never shown

























    4 Answers
    4






    active

    oldest

    votes








    4 Answers
    4






    active

    oldest

    votes









    active

    oldest

    votes






    active

    oldest

    votes









    2














    (1) ($supset$) ::
    begin{align*}
    A subset A cup B implies text{cl}(A) subset text{cl}(A cup B)
    \B subset A cup B implies text{cl}(B) subset text{cl}(A cup B)
    end{align*}
    therefore yielding that $text{cl}(A) cup text{cl}(B) subsettext{cl}(A cup B)$



    (2) ($subset$) ::



    The subset $text{cl}(A) cup text{cl}(B)$ is closed and both contains $A$ and $B$, therefore $A cup B subset text{cl}(A) cup text{cl}(B)$. $text{cl}(A cup B)$ is defined to be smallest closed set which contained $A cup B$, so that any closed set which contained $Acup B$ also contains $text{cl}(A cup B)$. Therefore $text{cl}(A cup B) subset text{cl}(A) cup text{cl}(B)$.






    share|cite|improve this answer





















    • Thanks for answer the proof makes sense but I still can't see what is wrong with my counterexample
      – TheGeometer
      Oct 26 '16 at 16:26










    • But how come $x$ in cl(A)? I have a neigbourhood of $x$ only intersecting at BA so this neigbourhood does not intersect A so it shouldn't be a limitpoint of A right?
      – TheGeometer
      Oct 26 '16 at 16:36










    • @TheGeometer Indeed, I've got confused! Define your two open sets to be $U_1$ and $U_2$. Then $U_1 cap U_2$ is a third open set containing $x$, that neither intersect $A$ or $B$. So $x notin text{cl}(A cup B)$ ;). So it's not a counterexample.
      – Hermès
      Oct 26 '16 at 16:44












    • Exactly! Thanks man I got so paranoid with that problem:P
      – TheGeometer
      Oct 26 '16 at 16:45
















    2














    (1) ($supset$) ::
    begin{align*}
    A subset A cup B implies text{cl}(A) subset text{cl}(A cup B)
    \B subset A cup B implies text{cl}(B) subset text{cl}(A cup B)
    end{align*}
    therefore yielding that $text{cl}(A) cup text{cl}(B) subsettext{cl}(A cup B)$



    (2) ($subset$) ::



    The subset $text{cl}(A) cup text{cl}(B)$ is closed and both contains $A$ and $B$, therefore $A cup B subset text{cl}(A) cup text{cl}(B)$. $text{cl}(A cup B)$ is defined to be smallest closed set which contained $A cup B$, so that any closed set which contained $Acup B$ also contains $text{cl}(A cup B)$. Therefore $text{cl}(A cup B) subset text{cl}(A) cup text{cl}(B)$.






    share|cite|improve this answer





















    • Thanks for answer the proof makes sense but I still can't see what is wrong with my counterexample
      – TheGeometer
      Oct 26 '16 at 16:26










    • But how come $x$ in cl(A)? I have a neigbourhood of $x$ only intersecting at BA so this neigbourhood does not intersect A so it shouldn't be a limitpoint of A right?
      – TheGeometer
      Oct 26 '16 at 16:36










    • @TheGeometer Indeed, I've got confused! Define your two open sets to be $U_1$ and $U_2$. Then $U_1 cap U_2$ is a third open set containing $x$, that neither intersect $A$ or $B$. So $x notin text{cl}(A cup B)$ ;). So it's not a counterexample.
      – Hermès
      Oct 26 '16 at 16:44












    • Exactly! Thanks man I got so paranoid with that problem:P
      – TheGeometer
      Oct 26 '16 at 16:45














    2












    2








    2






    (1) ($supset$) ::
    begin{align*}
    A subset A cup B implies text{cl}(A) subset text{cl}(A cup B)
    \B subset A cup B implies text{cl}(B) subset text{cl}(A cup B)
    end{align*}
    therefore yielding that $text{cl}(A) cup text{cl}(B) subsettext{cl}(A cup B)$



    (2) ($subset$) ::



    The subset $text{cl}(A) cup text{cl}(B)$ is closed and both contains $A$ and $B$, therefore $A cup B subset text{cl}(A) cup text{cl}(B)$. $text{cl}(A cup B)$ is defined to be smallest closed set which contained $A cup B$, so that any closed set which contained $Acup B$ also contains $text{cl}(A cup B)$. Therefore $text{cl}(A cup B) subset text{cl}(A) cup text{cl}(B)$.






    share|cite|improve this answer












    (1) ($supset$) ::
    begin{align*}
    A subset A cup B implies text{cl}(A) subset text{cl}(A cup B)
    \B subset A cup B implies text{cl}(B) subset text{cl}(A cup B)
    end{align*}
    therefore yielding that $text{cl}(A) cup text{cl}(B) subsettext{cl}(A cup B)$



    (2) ($subset$) ::



    The subset $text{cl}(A) cup text{cl}(B)$ is closed and both contains $A$ and $B$, therefore $A cup B subset text{cl}(A) cup text{cl}(B)$. $text{cl}(A cup B)$ is defined to be smallest closed set which contained $A cup B$, so that any closed set which contained $Acup B$ also contains $text{cl}(A cup B)$. Therefore $text{cl}(A cup B) subset text{cl}(A) cup text{cl}(B)$.







    share|cite|improve this answer












    share|cite|improve this answer



    share|cite|improve this answer










    answered Oct 26 '16 at 16:12









    Hermès

    1,809612




    1,809612












    • Thanks for answer the proof makes sense but I still can't see what is wrong with my counterexample
      – TheGeometer
      Oct 26 '16 at 16:26










    • But how come $x$ in cl(A)? I have a neigbourhood of $x$ only intersecting at BA so this neigbourhood does not intersect A so it shouldn't be a limitpoint of A right?
      – TheGeometer
      Oct 26 '16 at 16:36










    • @TheGeometer Indeed, I've got confused! Define your two open sets to be $U_1$ and $U_2$. Then $U_1 cap U_2$ is a third open set containing $x$, that neither intersect $A$ or $B$. So $x notin text{cl}(A cup B)$ ;). So it's not a counterexample.
      – Hermès
      Oct 26 '16 at 16:44












    • Exactly! Thanks man I got so paranoid with that problem:P
      – TheGeometer
      Oct 26 '16 at 16:45


















    • Thanks for answer the proof makes sense but I still can't see what is wrong with my counterexample
      – TheGeometer
      Oct 26 '16 at 16:26










    • But how come $x$ in cl(A)? I have a neigbourhood of $x$ only intersecting at BA so this neigbourhood does not intersect A so it shouldn't be a limitpoint of A right?
      – TheGeometer
      Oct 26 '16 at 16:36










    • @TheGeometer Indeed, I've got confused! Define your two open sets to be $U_1$ and $U_2$. Then $U_1 cap U_2$ is a third open set containing $x$, that neither intersect $A$ or $B$. So $x notin text{cl}(A cup B)$ ;). So it's not a counterexample.
      – Hermès
      Oct 26 '16 at 16:44












    • Exactly! Thanks man I got so paranoid with that problem:P
      – TheGeometer
      Oct 26 '16 at 16:45
















    Thanks for answer the proof makes sense but I still can't see what is wrong with my counterexample
    – TheGeometer
    Oct 26 '16 at 16:26




    Thanks for answer the proof makes sense but I still can't see what is wrong with my counterexample
    – TheGeometer
    Oct 26 '16 at 16:26












    But how come $x$ in cl(A)? I have a neigbourhood of $x$ only intersecting at BA so this neigbourhood does not intersect A so it shouldn't be a limitpoint of A right?
    – TheGeometer
    Oct 26 '16 at 16:36




    But how come $x$ in cl(A)? I have a neigbourhood of $x$ only intersecting at BA so this neigbourhood does not intersect A so it shouldn't be a limitpoint of A right?
    – TheGeometer
    Oct 26 '16 at 16:36












    @TheGeometer Indeed, I've got confused! Define your two open sets to be $U_1$ and $U_2$. Then $U_1 cap U_2$ is a third open set containing $x$, that neither intersect $A$ or $B$. So $x notin text{cl}(A cup B)$ ;). So it's not a counterexample.
    – Hermès
    Oct 26 '16 at 16:44






    @TheGeometer Indeed, I've got confused! Define your two open sets to be $U_1$ and $U_2$. Then $U_1 cap U_2$ is a third open set containing $x$, that neither intersect $A$ or $B$. So $x notin text{cl}(A cup B)$ ;). So it's not a counterexample.
    – Hermès
    Oct 26 '16 at 16:44














    Exactly! Thanks man I got so paranoid with that problem:P
    – TheGeometer
    Oct 26 '16 at 16:45




    Exactly! Thanks man I got so paranoid with that problem:P
    – TheGeometer
    Oct 26 '16 at 16:45











    2














    In your proposed counterexample, you've forgotten that open sets are closed under finite intersection.



    So if $x$ has a neighborhood that only meets $Acup B$ in $Bsetminus A$ and a neighborhood that only meets $Acup B$ in $Asetminus B$, then the intersection of these neighborhoods doesn't meet $Acup B$ at all.






    share|cite|improve this answer


























      2














      In your proposed counterexample, you've forgotten that open sets are closed under finite intersection.



      So if $x$ has a neighborhood that only meets $Acup B$ in $Bsetminus A$ and a neighborhood that only meets $Acup B$ in $Asetminus B$, then the intersection of these neighborhoods doesn't meet $Acup B$ at all.






      share|cite|improve this answer
























        2












        2








        2






        In your proposed counterexample, you've forgotten that open sets are closed under finite intersection.



        So if $x$ has a neighborhood that only meets $Acup B$ in $Bsetminus A$ and a neighborhood that only meets $Acup B$ in $Asetminus B$, then the intersection of these neighborhoods doesn't meet $Acup B$ at all.






        share|cite|improve this answer












        In your proposed counterexample, you've forgotten that open sets are closed under finite intersection.



        So if $x$ has a neighborhood that only meets $Acup B$ in $Bsetminus A$ and a neighborhood that only meets $Acup B$ in $Asetminus B$, then the intersection of these neighborhoods doesn't meet $Acup B$ at all.







        share|cite|improve this answer












        share|cite|improve this answer



        share|cite|improve this answer










        answered Oct 26 '16 at 16:43









        Alex Kruckman

        26.7k22556




        26.7k22556























            1














            Let $xin Cl(Acup B)$ then every open set containing $x$ intersects $Acup B$. Thus for $x in U_alpha$ where $U_alpha$ is open in $X$. If $U_alpha$ intersects $A$, then $x in Cl(A)$ else $x in Cl(B)$ either way $x in Cl(A)cup Cl(B)$.






            share|cite|improve this answer





















            • This is because if $yin Cl(A)$ iff every open set containing $y$ intersects $A$.
              – Riemann-bitcoin.
              Oct 26 '16 at 16:10










            • I think I have a confusion with the definition. How I know it is that $x$ is a limit point of a subset $S$ of a topological space if every neigbourhood of $x$ intersects with $S$. So in our case if $x$ has a neigbourhood only intersecting $AB$ and another only intersecting $BA$ it should be a limit point only of $AcupB$ and not of A or B.Am I using wrond definition or saying something wrong?
              – TheGeometer
              Oct 26 '16 at 16:19










            • The $Cl(A)$ is the limit points unioned with the set A. So $Cl(A)=A cup A'$. I believe this is the confusion, though I am slightly confused on what your counterexample is saying.
              – Riemann-bitcoin.
              Oct 26 '16 at 16:57


















            1














            Let $xin Cl(Acup B)$ then every open set containing $x$ intersects $Acup B$. Thus for $x in U_alpha$ where $U_alpha$ is open in $X$. If $U_alpha$ intersects $A$, then $x in Cl(A)$ else $x in Cl(B)$ either way $x in Cl(A)cup Cl(B)$.






            share|cite|improve this answer





















            • This is because if $yin Cl(A)$ iff every open set containing $y$ intersects $A$.
              – Riemann-bitcoin.
              Oct 26 '16 at 16:10










            • I think I have a confusion with the definition. How I know it is that $x$ is a limit point of a subset $S$ of a topological space if every neigbourhood of $x$ intersects with $S$. So in our case if $x$ has a neigbourhood only intersecting $AB$ and another only intersecting $BA$ it should be a limit point only of $AcupB$ and not of A or B.Am I using wrond definition or saying something wrong?
              – TheGeometer
              Oct 26 '16 at 16:19










            • The $Cl(A)$ is the limit points unioned with the set A. So $Cl(A)=A cup A'$. I believe this is the confusion, though I am slightly confused on what your counterexample is saying.
              – Riemann-bitcoin.
              Oct 26 '16 at 16:57
















            1












            1








            1






            Let $xin Cl(Acup B)$ then every open set containing $x$ intersects $Acup B$. Thus for $x in U_alpha$ where $U_alpha$ is open in $X$. If $U_alpha$ intersects $A$, then $x in Cl(A)$ else $x in Cl(B)$ either way $x in Cl(A)cup Cl(B)$.






            share|cite|improve this answer












            Let $xin Cl(Acup B)$ then every open set containing $x$ intersects $Acup B$. Thus for $x in U_alpha$ where $U_alpha$ is open in $X$. If $U_alpha$ intersects $A$, then $x in Cl(A)$ else $x in Cl(B)$ either way $x in Cl(A)cup Cl(B)$.







            share|cite|improve this answer












            share|cite|improve this answer



            share|cite|improve this answer










            answered Oct 26 '16 at 16:06









            Riemann-bitcoin.

            336111




            336111












            • This is because if $yin Cl(A)$ iff every open set containing $y$ intersects $A$.
              – Riemann-bitcoin.
              Oct 26 '16 at 16:10










            • I think I have a confusion with the definition. How I know it is that $x$ is a limit point of a subset $S$ of a topological space if every neigbourhood of $x$ intersects with $S$. So in our case if $x$ has a neigbourhood only intersecting $AB$ and another only intersecting $BA$ it should be a limit point only of $AcupB$ and not of A or B.Am I using wrond definition or saying something wrong?
              – TheGeometer
              Oct 26 '16 at 16:19










            • The $Cl(A)$ is the limit points unioned with the set A. So $Cl(A)=A cup A'$. I believe this is the confusion, though I am slightly confused on what your counterexample is saying.
              – Riemann-bitcoin.
              Oct 26 '16 at 16:57




















            • This is because if $yin Cl(A)$ iff every open set containing $y$ intersects $A$.
              – Riemann-bitcoin.
              Oct 26 '16 at 16:10










            • I think I have a confusion with the definition. How I know it is that $x$ is a limit point of a subset $S$ of a topological space if every neigbourhood of $x$ intersects with $S$. So in our case if $x$ has a neigbourhood only intersecting $AB$ and another only intersecting $BA$ it should be a limit point only of $AcupB$ and not of A or B.Am I using wrond definition or saying something wrong?
              – TheGeometer
              Oct 26 '16 at 16:19










            • The $Cl(A)$ is the limit points unioned with the set A. So $Cl(A)=A cup A'$. I believe this is the confusion, though I am slightly confused on what your counterexample is saying.
              – Riemann-bitcoin.
              Oct 26 '16 at 16:57


















            This is because if $yin Cl(A)$ iff every open set containing $y$ intersects $A$.
            – Riemann-bitcoin.
            Oct 26 '16 at 16:10




            This is because if $yin Cl(A)$ iff every open set containing $y$ intersects $A$.
            – Riemann-bitcoin.
            Oct 26 '16 at 16:10












            I think I have a confusion with the definition. How I know it is that $x$ is a limit point of a subset $S$ of a topological space if every neigbourhood of $x$ intersects with $S$. So in our case if $x$ has a neigbourhood only intersecting $AB$ and another only intersecting $BA$ it should be a limit point only of $AcupB$ and not of A or B.Am I using wrond definition or saying something wrong?
            – TheGeometer
            Oct 26 '16 at 16:19




            I think I have a confusion with the definition. How I know it is that $x$ is a limit point of a subset $S$ of a topological space if every neigbourhood of $x$ intersects with $S$. So in our case if $x$ has a neigbourhood only intersecting $AB$ and another only intersecting $BA$ it should be a limit point only of $AcupB$ and not of A or B.Am I using wrond definition or saying something wrong?
            – TheGeometer
            Oct 26 '16 at 16:19












            The $Cl(A)$ is the limit points unioned with the set A. So $Cl(A)=A cup A'$. I believe this is the confusion, though I am slightly confused on what your counterexample is saying.
            – Riemann-bitcoin.
            Oct 26 '16 at 16:57






            The $Cl(A)$ is the limit points unioned with the set A. So $Cl(A)=A cup A'$. I believe this is the confusion, though I am slightly confused on what your counterexample is saying.
            – Riemann-bitcoin.
            Oct 26 '16 at 16:57













            0














            Infinite point sequence from (A ∪ B) contains an infinite subsequence from A or contains an infinite subsequence from B






            share|cite|improve this answer





















            • ok but what is wrong wit my counterexample?
              – TheGeometer
              Oct 26 '16 at 16:04
















            0














            Infinite point sequence from (A ∪ B) contains an infinite subsequence from A or contains an infinite subsequence from B






            share|cite|improve this answer





















            • ok but what is wrong wit my counterexample?
              – TheGeometer
              Oct 26 '16 at 16:04














            0












            0








            0






            Infinite point sequence from (A ∪ B) contains an infinite subsequence from A or contains an infinite subsequence from B






            share|cite|improve this answer












            Infinite point sequence from (A ∪ B) contains an infinite subsequence from A or contains an infinite subsequence from B







            share|cite|improve this answer












            share|cite|improve this answer



            share|cite|improve this answer










            answered Oct 26 '16 at 16:00









            kotomord

            1,460626




            1,460626












            • ok but what is wrong wit my counterexample?
              – TheGeometer
              Oct 26 '16 at 16:04


















            • ok but what is wrong wit my counterexample?
              – TheGeometer
              Oct 26 '16 at 16:04
















            ok but what is wrong wit my counterexample?
            – TheGeometer
            Oct 26 '16 at 16:04




            ok but what is wrong wit my counterexample?
            – TheGeometer
            Oct 26 '16 at 16:04


















            draft saved

            draft discarded




















































            Thanks for contributing an answer to Mathematics Stack Exchange!


            • Please be sure to answer the question. Provide details and share your research!

            But avoid



            • Asking for help, clarification, or responding to other answers.

            • Making statements based on opinion; back them up with references or personal experience.


            Use MathJax to format equations. MathJax reference.


            To learn more, see our tips on writing great answers.





            Some of your past answers have not been well-received, and you're in danger of being blocked from answering.


            Please pay close attention to the following guidance:


            • Please be sure to answer the question. Provide details and share your research!

            But avoid



            • Asking for help, clarification, or responding to other answers.

            • Making statements based on opinion; back them up with references or personal experience.


            To learn more, see our tips on writing great answers.




            draft saved


            draft discarded














            StackExchange.ready(
            function () {
            StackExchange.openid.initPostLogin('.new-post-login', 'https%3a%2f%2fmath.stackexchange.com%2fquestions%2f1986224%2fclosure-of-the-union-union-of-closures%23new-answer', 'question_page');
            }
            );

            Post as a guest















            Required, but never shown





















































            Required, but never shown














            Required, but never shown












            Required, but never shown







            Required, but never shown

































            Required, but never shown














            Required, but never shown












            Required, but never shown







            Required, but never shown







            Popular posts from this blog

            An IMO inspired problem

            Management

            Has there ever been an instance of an active nuclear power plant within or near a war zone?